跳到主要內容

發表文章

目前顯示的是 2012的文章

[數學分析] 隱函數定理

在開始之前我們先說明到底 隱函數定理 想解決什麼問題?也就是: 何時能把 多變數函數 $f(x,y)=0$ 中的變數 用另一個變數表示 e.g., $x$ 用 $y$ 表示。 考慮 $f$ 為 雙變數函數 且 $f \in C^1$,則函數 $f$ 在 點 $(a,b)$ 滿足 \[ f(a,b) = 0,\;\; \frac{\partial f}{\partial y}(a,b) \neq 0 \]則在 $(a,b)$ 附近的鄰域之內,我們可求解方程式 $f(x,y) =0$ 並將 $y$ 用 $x$ 表示。 同理,若在 \[ f(a,b) = 0,\;\; \frac{\partial f}{\partial x}(a,b) \neq 0 \]則我們就可在  $(a,b)$ 附近的鄰域之內,我們可求解方程式 $f(x,y) =0$ 中的 $x$ 用 $y$ 表示。 Example 考慮 $f(x,y) := x^2 + y^2 -1$ 。 Q1: 求解 $f(x,y)=0$ 在 $(x,y)=(a,b) = (1/\sqrt{2}, 1/\sqrt{2})$處,問是否其解 $x$ 可用 $y$ 表示 or  將解 $y$ 用 $x$ 表示? Q2: 求解 $f(x,y)=0$ 在 $(x,y)=(a,b) = (1, 0)$處,問是否其解 $x$ 可用 $y$ 表示 or  將解 $y$ 用 $x$ 表示? Proof: 考慮 $(a,b)= (1/\sqrt{2}, 1/\sqrt{2})$ 且觀察 \[\left\{ \begin{array}{l} \frac{{\partial f}}{{\partial y}}\left( {a,b} \right) = {\left. {2y} \right|_{\left( {a,b} \right)}} = 2b = \frac{1}{{\sqrt 2 }} \ne 0\\ \frac{{\partial f}}{{\partial x}}\left( {a,b} \right) = {\left. {2x} \right|_{\left( {a,b} \right)}} = 2a = \frac{1}{{\sqrt 2 }} \ne 0 \end{array} \r

[系統理論] Fourier Transform and Laplace Transform

我們首先看看 雙邊形拉氏轉換 (Bilateral Laplace transform): 對訊號 $x(t)$ 我們定義 Bilateral Laplace transform : \[ X(s) := \int_{-\infty}^{\infty} x(t) e^{-st} dt \]且 $s = \sigma + j \omega$ 為 complex variable 。 注意到如果我們令 $\sigma =0$,亦即 $s = j \omega$ (purly imaginary), 則上式變為 \[ X(s)|_{s=j \omega} = X(j\omega ) = \int_{{-\infty }}^\infty  x (t){e^{ - j\omega t}}dt \] 上式即為 Fourier Transform。 事實上, Laplace transform 亦與 Fourier transform 可以有更直接的關係,現在我們讓 $s$ 變回原本的 complex variable 形式: $s = \sigma + j \omega$ 並代回 Laplace transform 我們可得 \[\begin{array}{l} {\left. {X\left( s \right)} \right|_{s = \sigma  + j\omega }} = X\left( {\sigma  + j\omega } \right) = \int_{ - \infty }^\infty  {x\left( t \right){e^{ - \left( {\sigma  + j\omega } \right)t}}dt} \\ \begin{array}{*{20}{c}} {}&{}&{}&{}&{}&{} \end{array} = \int_{ - \infty }^\infty  {\left[ {x\left( t \right){e^{ - \sigma t}}} \right]{e^{ - j\omega t}}dt} \end{array}\]我們可以觀察到上式為 $x(t) e^{\sigma t}$ 的 Fourier transform,亦即我們可以把 $

[分享] 淺談歸正神學

甚麼是歸正神學 (Reformed Theology)? 基督徒身在這個世代,應該能更清楚而明確的信仰,也就是 "應當更明白的知道深知我所信的這一位到底是誰? 因為經上記著  萬軍之耶和華吩咐:「你們務要認識我」  (何西阿書 6:3),筆者期盼能彼此勉勵,更加裝備自己,在遇到各種挑戰時候,能用神的道站穩腳步。 歸正神學  歸正神學在 堅決相信凡事應當  "以神為本"  而非 "以人為本",相信全本聖經都是上帝默示的,是神所啟示的。沒有任何錯誤。並且不斷的回歸聖經真道的神學。 這種神學 與 部分當今主流神學 強調只相信聖經一部分,否定 其餘經上教導,或者透過引入心理學/現代科學來解釋聖經,或者採用靈恩運動,高舉內在醫治,方言禱告的手法大相逕庭。 歸正神學的 真正要義 由   加爾文 ( Jean Calvin )  提出五點要義:稱為TULIP(為鬱金香之義)。 1.全然敗壞(Total depravity) 人類由於 始祖亞當 的墮落而無法以自己的能力作任何靈性上的善事。 2.無條件的揀選(Unconditional election) 上帝對於罪人揀選是無條件的,祂 的揀選並非因為人在倫理道德上的優點,也非 祂 預見了人將發生的信心。也就是 揀選與救贖之功 全然在於上帝,人無法有所作為。 3.有限的代贖(Limited atonement) 基督釘十字架只是為那些預先蒙選之人,不是為世上所有的人。 4.不可抗拒的恩典(Irresistible grace) 人不可能拒絕 上帝 的救恩, 上帝 拯救人的恩典不可能因為人的原因而被阻撓,亦無法被人拒絕。 5.聖徒恆忍蒙保守(Perseverance of the saints) 參考資料[基督教小小羊園地] http://blog.roodo.com/yml/archives/2635383.html 求主的道光照我們,指導我們分辨甚麼才是真理。因為 你們必曉得真理,真理必叫你們得以自由(約翰福音8:32)

[系統理論] 閉迴路系統的暫態響應 與 eigenvalue/eigenvector 關係

首先考慮  $n$階 線性微分方程如下 \[ \dot {\bf x}(t) = G{\bf x}(t); \;\; { \bf x}(0) ={\bf x}_0 \]上式一般表示為 無外力輸入 $u$ 的系統。且我們可對其求解得到 \[{\bf x}(t) = e^{Gt} {\bf x}_0 \ \ \ \ \ \ (*) \] 我們可進一步將上式的解 $(*)$ 重新用 矩陣 $G$ 的 eigenvalues/eigenvectors 表示; i.e., 若 $G$ 為 $n \times n$ 則 下式 eigenvalue-eigenvector 關係需被滿足 \[G{{\bf{v}}_i} = {\lambda _i}{{\bf{v}}_i}, \text{ for $i=1,2,...,n$} \]其中 $\lambda_i$ 為 $G$ 的 eigenvalues 且 $v_i$ 為對應的 eigenvectors。 注意到在此我們假設 $G$ 的 eigenvalues 均相異。現在使用這些 eigenvectors, $v_i$,建構  非奇異轉換矩陣 或稱 modal matrix  $M$ 如下 \[M: = \left[ {\begin{array}{*{20}{c}} {{{\bf{v}}_1}}&{{{\bf{v}}_2}}& \cdots &{{{\bf{v}}_n}} \end{array}} \right] \]注意到 $M$ 為 nonsingular 因為 $\{v_i\}$ 彼此線性獨立 (因為相異 eigenvalue 對應 線性獨立的 eigenvector) ;現在使用 $M$,定義下列新狀態 $\bf z$ 轉換 \[ {\bf x} = M {\bf z} \]將上述新的狀態關系代入原系統 $\dot {\bf x}(t) = G{\bf x}(t)$ 我們可以改寫如下 \[\begin{array}{l} {\bf{\dot x}}(t) = G{\bf{x}}(t)\\  \Rightarrow M{\bf{\dot z}}\left( t \right) = GM{\bf{z}}(t)\\  \Rightarrow {\bf{\dot z}}\le

[控制理論] 狀態回授控制(1)- Eigenstructure Assignment

控制理論中最重要的本質 便是 回授控制 (feedback control),在實現上,回授控制具有下列四種主要功能: 改善/保證 系統穩定度 降低系統的 敏感度(提升強健性) 改善系統 抑制 低頻外在干擾 或者 抑制 高頻雜訊 的能力 改善 系統暫態響應 而 Eigenstructure Assignment (同時給定 eigenvalue 與 eigenvector )主要是透過回授控制達成第四個目標:改善 系統的 暫態響應。以下我們介紹 全狀態回授 (Full State Feedback) 的 Eigenstructure Assignment。 考慮系統 \[\dot x\left( t \right) = Ax\left( t \right) + Bu\left( t \right) \] 其中 $x(t) \in \mathbb{R}^n$; $u(t) \in \mathbb{R}^m$ 且狀態回授控制器 $u(t) = F x(t)$。 則將控制器帶入系統,可得閉迴路系統如下 \[\dot x\left( t \right) = \left( {A + BF} \right)x\left( t \right) \] 現在令 $\lambda_i$ 為系統 $A+BF$ 的 eigenvalue,且 $v_i$ 為對應的 eigenvector,則我們有 eigenvalue-eigenvector relationship 如下 \[ (A+BF) v_i = \lambda_i v_i \] 上式可改寫為 \[\begin{array}{l} (A + BF){v_i} = {\lambda _i}{v_i}\\  \Rightarrow \left( {{\lambda _i}I - A} \right){v_i} - BF{v_i} = 0 \end{array}\] 故我們現在定義 $S_{\lambda_i }:= [\lambda_i I - A \;\; B]$ 且定義其對應的分割矩陣 \[{K_{{\lambda _i}}}: = \left[ {\begin{array}{*{20}{c}} {{N_{{\lambda _i}}}}\\ {{M_{{\lambda

[機率論] Almost Sure Convergence

固定 機率空間 $(\Omega, \mathcal{B}, P)$。我們說 隨機事件 $A$  almost surely (a.s.) 成立,若下列條件成立: 存在一個事件 $N \in \mathcal{B}$ 且 $P(N)=0$ 使得 若 $\omega \in N^c$ 事件 $A$ 都成立。 Example 1: Two r.v. equal Almost Surely 令 $X, X'$ 為兩個隨機變數,則 $X=X'$ almost surely 亦即 \[ P(\omega: X(\omega) = X'(\omega)) = 1 \]亦即,存在事件 $N \in \mathcal{B}$ 使得 $P(N)=0$ 且  $\omega \in N^c \Rightarrow X(\omega) = X(\omega)'$ 都成立。$\square$ Example 2:  令 $X, X'$ 為兩個隨機變數,則 $X \le X'$ almost surely 亦即 \[ P(\omega: X(\omega) \le X'(\omega)) = 1 \]亦即,存在事件 $N \in \mathcal{B}$ 使得 $P(N)=0$ 且  $\omega \in N^c \Rightarrow X(\omega) \le X(\omega)'$ 都成立。$\square$ Example 3: random variable sequence 若' $\{X_n\}$ 為 隨機變數的 sequence,則 $\lim_{n\rightarrow \infty}X_n$存在 almost surly 意指 存在事件 $N \in \mathcal{B}$ 使得 $P(N)=0$ 且  $\omega \in N^c \Rightarrow \lim_{n\rightarrow \infty}X_n$ 存在 ;此陳述等價為 \[ \limsup_{n \rightarrow \infty} X_n(\omega) = \liminf_{n\rightarrow \infty }X_n(\omega) \]我們會寫作 $\lim_{n \

[繪圖] 喜帖

by Chung-Han Hsieh

[系統理論] 連續時間 週期訊號的 Fourier Transform Representation

延續前篇 [系統理論] 連續時間 非週期訊號的 Fourier Transform Representation ,我們知道 非週期訊號$x(t)$ 若滿足 Dirchlet conditions 則 Fourier Transform 存在,且我們可寫成 \[\left\{ \begin{array}{l} X\left( {j\omega } \right) = \int_{ - \infty }^\infty  {x\left( t \right){e^{ - j\omega t}}dt} \ \ \ \  (1) \\ x\left( t \right) = \frac{1}{{2\pi }}\int_{ - \infty }^\infty  {X\left( {j\omega } \right){e^{j\omega t}}d\omega} \ \ \ \ (2) \end{array} \right.\]上式中 $(1)$ 稱為 $x(t)$ 的 Fourier Transform,$(2)$ 稱為 Inverse Fourier Transform。 事實上,對於週期訊號而言,除了 Fourier Series 之外, 我們亦可對 週期訊號 求解 Fourier Transform 。 那麼該怎麼做呢? 想法: 透過 Impulse function 幫助我們對 Fourier Transform 進行"取樣" 首先我們先做個觀察如下: 考慮一個訊號 $x(t)$ 其 Fourier Transform $X(j \omega)$ 為落在 $\omega = \omega_0$ 且面積為 $2 \pi$ 的單位脈衝函數如下 \[ X(j \omega) = 2 \pi \delta(\omega - \omega_0) \]現在利用 Inverse Fourier Transform 我們可得回 $x(t)$ \[\begin{array}{l} x\left( t \right) = \frac{1}{{2\pi }}\int_{ - \infty }^\infty  {X\left( {j\omega } \right){e^{j\omega t}}d\omega} \\ \begin{arr

[系統理論] 連續時間 非週期訊號的 Fourier Transform Representation

先前我們提及 對於 週期訊號 可以透過 Fourier Series Represenation,但如果要處理的對象是 非週期訊號 (aperiodic) 該怎麼辦呢? ------------------ 基本想法: 設法讓 非週期訊號 用 週期訊號表示 ,則原本對 週期訊號的 Fourier Series 展開的方法仍然適用。那麼要如何才能辦到? 我們讓非週期訊號以週期 $T \rightarrow \infty$ 的方式重現 週期訊號。 ------------------- 下面我們更具體一點的來看看如何實現上述的基本想法,現在考慮 $x(t)$ 為 (有限範圍) 的 連續 非週期訊號如下圖 亦即存在實數 $T_1$ 使得非週期訊號 $x(t) := 0$ 若 $|t| > T_1$。 現在我們複製上面的 有限範圍 ($-T_1 < t < T_1 $) 非週期訊號 $x(t)$,並藉此建構一週期為 $T$ 的 週期訊號 $\tilde{ x}(t)$ 如下: 注意到上圖我們所建構的週期訊號,對 $|t| < T/2 $, $\tilde{x}(t) \equiv x(t)$。且對於上述週期訊號 $\tilde {x} (t)$,我們有 Fourier Series Pair 如下 \[\left\{ \begin{array}{l} \tilde x\left( t \right) = \sum\limits_{k =  - \infty }^\infty  {{a_k}{e^{jk{\omega _0}t}}} \\ {a_k} = \frac{1}{T}\int_{ - T/2}^{T/2} {\tilde x\left( t \right)} {e^{ - jk{\omega _0}t}}dt \end{array} \right.\] 其中 $\omega_0 = 2\pi/T$。 現在注意到因為  $\tilde{x}(t) \equiv x(t)$ 只有在 $|t| < T/2$ 成立,且對於 $|t| \ge T/2$ 而言, $x(t) =0$,也就是說 \[\tilde x\left( t \right) = \left\{ \begin{array}{l}

[系統理論] 連續時間週期訊號的 Fourier Series Representation (4)-Parseval's relation for periodic signal

令 $x(t)$ 與 $y(t)$ 為具有週期為 $T$ 的連續時間週期訊號,且存在 Fourier Series Representation 如下 \[\begin{array}{l} x\left( t \right) = \sum\limits_{k =  - \infty }^\infty  {{a_k}} {e^{jk{\omega _0}t}}\\ y\left( t \right) = \sum\limits_{k =  - \infty }^\infty  {{b_k}} {e^{jk{\omega _0}t}} \end{array} \] 我們首先證明下面的結果: Fact: 給定兩時域訊號相乘 $\Rightarrow$ 頻域訊號 convolution 定義兩訊號乘積 $z(t) := x(t) y(t) = \sum\limits_{k =  - \infty }^\infty  {{c_k}} {e^{jk{\omega _0}t}}$,則 其乘積的 Fourier Series Coefficient 為離散 convolution \[ c_k = \sum_{n=-\infty}^{\infty}a_n b_{k-n} \] Proof: 觀察 $z(t) = x(t)y(t)$ 如下 \[\begin{array}{l} x\left( t \right)y\left( t \right) = \sum\limits_{n =  - \infty }^\infty  {{a_n}} {e^{jn{\omega _0}t}}\sum\limits_{k =  - \infty }^\infty  {{b_k}} {e^{jk{\omega _0}t}}\\ \begin{array}{*{20}{c}} {}&{}&{}&{} \end{array} = \sum\limits_{k =  - \infty }^\infty  {\left( {{b_k}\sum\limits_{n =  - \infty }^\infty  {{a_n}} {e^{jn{\omega _0}t}}} \right){e^{jk{\omega _0}t}}} \end{array}\

[系統理論] 連續時間週期訊號的 Fourier Series Representation (3)- Convergence condition

令 $x(t)$ 為週期訊號, 若 $x(t)$ 存在 Fourier Series Representation,則我們可寫下 \[ x(t) = \sum_{k=-\infty}^{\infty} a_k e^{j k \omega_0 t} \] Fourier 認為 "任意" 週期訊號都可以被表示成 complex exponentials 的線性組合,亦即任意週期訊號都 存在 Fourier Series, 但事實上這個陳述並 不正確 。亦即,並非所有的週期訊號都有 Fourier Series;最關鍵的問題是 Fourier Series 本身牽涉到無窮級數,一旦級數涉及無窮項必然存在級數收斂性問題。 Validity of Fourier Series Representation 首先觀察一個具有週期 $T$ 的週期訊號 $x(t)$ ,但我們僅透過 有限 $N$ 項 complex exponentials 的線性組合來表示近似此週期訊號。用有限 $N$ 項 complex exponentials 線性組合的近似週期訊號記做 $x_N(t)$ 如下 \[{x_N}\left( t \right) = \sum\limits_{k =  - N}^N {{a_k}{e^{jk{\omega _0}t}}} \] 現在我們定義 $e_N(t)$ 為  $x(t)$ 與 $x_N(t)$的近似誤差 如下: \[{e_N}\left( t \right): = x\left( t \right) - {x_N}\left( t \right) = x\left( t \right) - \sum\limits_{k =  - N}^N {{a_k}{e^{jk{\omega _0}t}}} \]那麼有了上述 近似誤差 $e_N(t)$ 的定義,我們仍然不容易知道到底 $e_N(t)$ 怎樣算是好 ($x_N(t)$ 有多接近 $x(t)$)。故我們首先只觀察一個週期,然後透過 2-norm (或者說 energy idea) 定義 "近似誤差的大小" \[ E_N := \int_T |e_N(t)|^2dt \]那麼我們的目標是找出到底怎樣的 $a_k$ 使得 $E_N$ 被最

[電子學] 淺談雙極性接面電晶體(BJTs) 的基本想法。

真實電路中,常見的兩端點元件 e.g., 電阻、電感、電容、或者 二極體 (diode),但這些元件並無法將輸出端的電流或者電壓進行放大。故我們需要一個電路元件 可以幫助我們辦到這項目標。 比如說現在要建構一個新元件具備 電流控制 的電流放大器(current amplifier)元件。那麼這個元件應該具備怎樣的特性呢?? 我們首先繪製下圖 中間的 問號方塊即為我們要設計的三端子元件。目的是要透過電流 $i_s$ 來得到放大的輸出電流 $i_o$ 。現在我們把方塊內部繪製如下: 上圖中 $R_L$ 表示負載 (e.g., 馬達)。途中藍色線框標示處表示我們要設計的電流放大元件,觀察內部會發現有幾個設計參數待定: 輸入電阻 $R_i$ 輸出電阻 $R_o$ 放大倍數 $A$ 由於 $A$ 表示 放大倍數,我們會希望 $i_i$ 被放大 $A$ 倍; i.e., $A i_i$。 現在觀察上圖左方電路,計算 $i_i$ 電流:由分流定理我們可知 \[ i_i = i_s \frac{R_s}{R_s + R_i} \]觀察上式,若輸入電阻 $R_i$ 設計成很小,則我們可以得到 $i_i \cong i_s$ ($i_s$ 都流經電阻 $R_i$ ,沒有損失太多電流)。 現在觀察上圖右方電路,此時輸入到右方電路的電流為 $A i_i$,我們可以計算 輸出電流 $i_o$,由分流定理可知 \[ i_o = (A i_i) \cdot \frac{R_o}{R_o + R_L} \]故若 $R_o$ 選定很大,則我們可以得到較大輸出電流 $i_o$。 結論:對於電流放大元件的需求: 輸入電阻 $R_i$ 要小 輸出電組 $R_o$ 要大 放大倍數 $A$ 要大 ($\neq 0$) 那麼我們如何滿足上述條件呢? 透過 diode 單向導通的想法即可達成:首先取兩組diode,現在對其中一組 diode順向偏壓 (diode導通,此時等同得到輸入電阻很小),且對另外一組diode逆向偏壓 (diode不通,等同輸出電阻很大的效果),現在將兩組diode合併。如下圖 npn 電晶體: 另外亦可接成 npn 電晶體,將 Forward bias 換成 reverse bias 即可。

[數學分析] Volterra integral 與 Contraction Principle

首先回憶 Contraction Principle。 Theorem: Contraction Principle 若 $(X,d)$ 為 complete metric space 且 $\Phi$ 為 contraction on $X$,則 $\Phi$ 有  唯一  不動點 $x^*$ (unique fixed point);亦即 存在 $x^* \in X$ 使得 $\Phi(x^*) = x^*$ 考慮 $K$ 為在 $[a,b]\times[a,b]$上 連續函數 現在我們定義 Volterra Integral $\cal K$ 如下 若 $\phi \in C([a,b])$,$\cal K: C([a,b]) \to C([a,b])$ 且滿足 \[ \mathcal{K}f(x):= \phi(x) + \lambda \int_a^x K(x,y)f(y)dy \] 利用 Contraction Principle 我們可以證明下面命題: Proposition: 對任意 $\lambda>0$,Volterra Integral $\cal K$ 有 fixed point on $C([a,b])$ with sup metric。 Proof: 我們要證明 Volterra Integral $\cal K$ 有 fixed point on $C([a,b])$,由於 $C([a,b])$ with sup metric 為 complete metric space,故若我們可以證明  $\cal K$ 為 contraction 則由  Contraction Principle  可知必有唯一不動點。 回憶 一個函數 $\Phi$ 為 metric space $(X,d)$ 上的 contraction 定義為:$\Phi:X \to X$ 且 存在 $c$ 滿足 $0 \le c <1$ 使得 $d(\Phi(x),\Phi(y)) \le c \cdot d(x,y)$。 故我們觀察 \[\begin{array}{l} d(K{f_1},K{f_2}) = \left\| {K{f_1} - K{f_2}} \right\|\\ \begin{array

[線性系統] 動態方程式的求解(3) - LTV state equation- Total Solution

延續前篇文章  [線性系統] 動態方程式的求解(2) - LTV state equation- Homogeneous solution ,這次要介紹線性時變 (Linear Time Varying, LTV ) 系統的狀態方程的全解。 考慮下列 LTV 動態系統 \[\left\{ {\begin{array}{*{20}{l}} {{\bf{\dot x}}\left( t \right) = {\bf{A}}\left( t \right){\bf{x}}\left( t \right) + {\bf{B}}\left( t \right){\bf{u}}\left( t \right)}\\ {{\bf{y}}\left( t \right) = {\bf{C}}\left( t \right){\bf{x}}\left( t \right) + {\bf{D}}\left( t \right){\bf{u}}\left( t \right)} \end{array}} \right. \] 且假設  ${\bf{A}}\left( t \right)$ 為 $n \times n$ 且矩陣中每一項元素 都為對時間 $t$ 連續函數。 Comment: 1. 上式中 ${{\bf{\dot x}}\left( t \right) = {\bf{A}}\left( t \right){\bf{x}}\left( t \right) + {\bf{B}}\left( t \right){\bf{u}}\left( t \right)}$ 稱為狀態方程 (State equation) 2. ${{\bf{y}}\left( t \right) = {\bf{C}}\left( t \right){\bf{x}}\left( t \right) + {\bf{D}}\left( t \right){\bf{u}}\left( t \right)}$ 稱為 輸出方程 (Output equation) =================== Claim: 給定初始狀態 ${\bf{x}}\left( {{t_0}} \right)$ 與 輸入 ${{\bf{u}}\left( t \right)}$,則狀態方程 ${{\bf{\d

[線性系統] 動態方程式的求解(2) - LTV state equation- Homogeneous solution

這次要介紹線性時變 (Linear Time Varying, LTV ) 系統的狀態方程求解。 考慮下列 LTV 動態系統 \[\left\{ {\begin{array}{*{20}{l}} {{\bf{\dot x}}\left( t \right) = {\bf{A}}\left( t \right){\bf{x}}\left( t \right) + {\bf{B}}\left( t \right){\bf{u}}\left( t \right)}\\ {{\bf{y}}\left( t \right) = {\bf{C}}\left( t \right){\bf{x}}\left( t \right) + {\bf{D}}\left( t \right){\bf{u}}\left( t \right)} \end{array}} \right. \] 且假設  ${\bf{A}}\left( t \right)$ 為 $n \times n$ 且矩陣中每一項元素 都為對時間 $t$ 連續函數。 NOTE: 若上述對 ${\bf{A}}\left( t \right)$ 時變矩陣的連續性假設成立,則對任意初始狀態 ${\bf{x}}\left( {{t_0}} \right)$ 與任意輸入 ${{\bf{u}}\left( t \right)}$, 狀態方程有唯一解。 (Proof ommitted) 在我們進行求解之前,我們首先求解 \[ {{\bf{\dot x}}\left( t \right) = {\bf{A}}\left( t \right){\bf{x}}\left( t \right)} \] 其中 ${\bf{A}}\left( t \right)$ 為 $n \times n$ 且每一個 entry 都為 對時間 $t$ 連續的函數。故對任意初始狀態 ${\bf{x}}_i\left( {{t_0}} \right)$  狀態方程存在唯一解 $ {{\bf{x}}_i}\left( t \right),\forall i = 1,2,...,n$ 。 我們可以將這些 $n$ 個解蒐集起來寫作矩陣形式如下: \[{\bf{X}}\left( t \right): = \left[ {\begin{arr